Warum sind die Zahlen Q im Raum R weder offen noch abgeschlossen?

4 Antworten

Vom Fragesteller als hilfreich ausgezeichnet

Hallo.

Der Beweis muss in 2 Richtungen erfolgen - jeweils als indirekter Beweis, (ich vermute, dass es leichter ist):

1. Richtung:

Annahme Q ist in R abgeschlossen. Dann muss die Komplementärmenge (R\Q) offen in R sein.

Weil R\Q offen ist: Sei x aus R\Q und es sei epsilon>0 und für alle y aus R mit Betrag(x-y) < epsilon gilt; y ist aus R\Q. Die spannende Frage ist jetzt: Gibt es eine rationale Zahl, die in dem Intervall [x-epsilon,x+epsilon] liegt?

Sei n aus N mit (1/n)<epsilon … nennen wir dieses (1/n) mal „Schrittweite“. Damit ist das Intervall [x-epsilon,x+epsilon] mindestens doppelt so lang wie die Schrittweite.

Daraus folgt: Wenn ich von 0 beginnend in der Schrittweite auf das Intervall „los gehe“, werde ich das Intervall irgendwann treffen (das kann man noch schöner mathematisch formulieren). Also: Es gibt ein m aus N mit (m/n) liegt in dem Interval [x-epsilon,x+epsilon] --> und damit m/n ist aus Q. --> Und damit ist die Annahme falsch, --> d.h. Q ist nicht abgeschlossen in R.

Bleibt noch die andere Beweisrichtung … die mache ich morgen.

Schöne Grüße

also diese richtung ist nachvollziehbar und sieht schon mal gut aus. Dann fehlt ja nur noch die andere. Danke schön, schon mal. :-) schöne Grüße

0

weder offen ,weil es zu keiner rationalen Zahlen eine Umgebung gibt, die nur aus rationalen Zahlen besteht.

noch abgeschlossen:

Die Grenzwerte rationaler Zahlen sind nicht immer rational.

hat es vielleicht damit zu tun, dass die Mächtigkeit von Q abzählbar unendlich, die von R aber überabzählbar unendlich ist?

Jetzt die 2. Richtung.

Annahme Q ist in R offen.

Weil Q offen ist: Sei q aus Q und es sei epsilon>0 und für alle y aus R mit Betrag(x-y) < epsilon gilt; y ist aus Q. Die spannende Frage ist jetzt: Gibt es eine nicht-rationale Zahl, die in dem Intervall [x-epsilon,x+epsilon] liegt?

Sei n aus N mit (wurzel(2)/n)<epsilon … nennen wir dieses (wurzel(2)/n) mal wie "oben" „Schrittweite“. Damit ist das Intervall [x-epsilon,x+epsilon] mindestens doppelt so lang wie die Schrittweite.

Daraus folgt: Wenn ich von 0 beginnend in der Schrittweite auf das Intervall „los gehe“, werde ich das Intervall irgendwann treffen. Also: Es gibt ein m aus N mit (m*wurzel(2)/n) liegt in dem Interval [x-epsilon,x+epsilon]

Jetzt bleibt noch zu zeigen: m*wurzel(2)/n ist NICHT aus Q.

Auch das mache ich wieder indirekt:

Annahme m*wurzel(2)/n ist aus Q.

Dann gibt es ganze Zahlen p und q mit m*wurzel(2)/n = p/q.

Jetzt bringen wir m und n auf die andere Seite und erhalten:

wurzel(2)= (pn)/(qm) ... und das ist falsch

Damit haben wir eine nicht-rationale Zahl in dem Interval und damit ist Q nicht offen in R.

Schöne Grüße